LSAT and Law School Admissions Forum

Get expert LSAT preparation and law school admissions advice from PowerScore Test Preparation.

 Haleyeastham
  • Posts: 33
  • Joined: Aug 03, 2015
|
#19397
I have reviewed this question several times and I am still not catching what I'm missing- I am seeing that choices B and C do not work for the same reason. Can you please explain why answer choice C is wrong? Thank you!
 Clay Cooper
PowerScore Staff
  • PowerScore Staff
  • Posts: 241
  • Joined: Jul 03, 2015
|
#19404
HI Haley,

Thanks for your question!

This is a tough game. We have three fish species to select and two plant species.

Answer choice C is incorrect because it is possible, under the rules of the game, to select both W and Z.

Let's deal with Z first. Since Z is not mentioned in any of our rules, it is essentially a random, and selecting it (as this answer choice does) does not force us to select or deselect any other variables. That means, then, that if we can also select W without breaking the rules, this answer choice would work within the restrictions of the game and would therefore be incorrect.

From the third rule, we know that J > W. However, neither J nor W (nor, remember, Z) is mentioned in any other rule. Therefore, we still have enough variables remaining to fulfill the empty spots and make this answer choice work.

We could select J, G, and L for the fish, and W and Z for the plants - which fits within the rules of the game and makes this answer choice work (and therefore be incorrect).

I hope that this answer has clarified the situation for you. Remember to focus on random variables whenever possible - in this case, L and Z, which we can throw into this permutation without restricting ourselves much.

Does that help?
 Haleyeastham
  • Posts: 33
  • Joined: Aug 03, 2015
|
#19516
Yes thank you very much!

Get the most out of your LSAT Prep Plus subscription.

Analyze and track your performance with our Testing and Analytics Package.